Aufgabe 5 gelöst

This commit is contained in:
WieErWill 2022-03-05 15:40:32 +01:00
parent 5492ba656d
commit 7340b5b030
2 changed files with 27 additions and 3 deletions

Binary file not shown.

View File

@ -221,11 +221,28 @@ Erlaubte Hilfsmittel: eine math. Formelsammlung/Nachschlagwerk, ein handbeschrie
\begin{parts}
\part Für welche Partei sollte er sich entscheiden, um mit maximaler Wahrscheinlichkeit Bundeskanzler zu werden?
\begin{solution}
S: wird Spitzenkandidat, K: wird Bundeskanzler,
$P(A \cap S) = 0,1$, $P(B\cap S)=0,2$, $P(C\cap S)=1$
$P(A\cap K)= 0,6$, $P(B\cap K)=0,45$, $P(C\cap K)=0,02$
$P_A(S\cap K) = P(A \cap S) \cap P(A \cap K) = 0,1 * 0,6 = 0,06$
$P_B(S \cap K)= P(B \cap S) \cap P(B \cap K) = 0,2 * 0,45 = 0,09$
$P_C(C \cap K)= P(C \cap S) \cap P(C \cap K)= 1 * 0,02 = 0,02$
Markus hat bei Partei B die größten Chancen Bundeskanzler zu werden (mit 9\%).
\end{solution}
\part Markus lässt die Würfel entscheiden. Bei $1$ tritt er Partei $A$ bei, bei $2$ oder $3$ Partei $B$ und bei $4,5$ oder $6$ gründet er Partei $C$. Markus wird tatsächlich Bundeskanzler. Mit welcher Wahrscheinlichkeit hat er dann Partei $C$ gegründet.
\begin{solution}
$P(\text{Tritt A bei}) = \frac{1}{6}$, $P(\text{Tritt B bei})=\frac{2}{6}=\frac{1}{3}$, $P(\text{Tritt C bei})=\frac{3}{6}=\frac{1}{2}$
$P_{\text{Tritt C bei}}(\text{C gewinnt mit ihm}) = \frac{P_C(C \cap K)}{P(\text{Tritt C bei})} = \frac{0,02}{0,5} = 0,04$
Wenn Markus Bundeskanzler wird, hat er mit 4\% Wahrscheinlichkeit seine eigene Partei C gegründet.
\end{solution}
\end{parts}
@ -315,8 +332,15 @@ Erlaubte Hilfsmittel: eine math. Formelsammlung/Nachschlagwerk, ein handbeschrie
\end{center}
\end{solution}
\part Zeige, dass für jede natürliche Zahl $k\leq 1$ gilt: Jeder Baum, der eine Ecke vom Grad $k$ enthält, hat mindestens $k$ Blätter.
\part Zeige, dass für jede natürliche Zahl $k\geq 1$ gilt: Jeder Baum, der eine Ecke vom Grad $k$ enthält, hat mindestens $k$ Blätter.
\begin{solution}
Induktionsannahme: Es wird angenommen der Baum ist homogen verteilt, d.h. die Teilbäume jedes Baumes sind von gleicher Kantenlänge (Größe). Besitzt ein Teilbaum keinen Unterbaum, so ist er ein Blatt.
Induktionsstart: Für $k=1$ besitzt ein Baum maximal $2^1$ Kanten mit mindestens 1 Blatt. Daraus folgt $k=1=\sum Blätter$ stimmt
Induktionsschritt: Für $k=n+1$ besitzt ein Baum maxumal $2^{n+1}$ Kanten mit mindestens 1 Blatt. Daraus folgt $k=n+1...$
\end{solution}
\end{parts}
\end{questions}